Type 1 - Must be true/Conclusion/Inference correct answersWhich one of the following can be logically
inferred from the passage?
Type 1 - Must be true/Conclusion/Inference correct answersIf all of the _______ statements are true,
which one of the following must also be true?
Type 1 - Must be true/Conclusion/Inference correct answersWhich one of the following can be inferred
from the passage?
Type 1 - Must be true/Conclusion/Inference correct answersThe above statements, if true, most strongly
support which one of the following?
Type 1 - Must be true/Conclusion/Inference correct answersWhich one of the following is most strongly
supported by the information above?
Type 7 - Main Point correct answersWhich one of the following is the main conclusion of the zoo
directors argument?
Type 7 - Main Point correct answersThe main point of the argument is
Type 7 - Main Point correct answersWhich one of the following most accurately expresses the main
conclusion?
Type 8 - Role/Function correct answers________ plays what role in the argument?
Type 8 - Role/Function correct answersWhich one of the following most accurately describes the role
played?
, Type 3 - Weaken/Undermine correct answersWhich one of the following, if true, most seriously weakens
the argument?
Type 3 - Weaken/Undermine correct answersEach of the following, if true, weakens the argument
EXCEPT...
Type 3 - Weaken/Undermine correct answersWhich one of the following, if true, casts the most doubt on
the conclusion above?
Type 2 - Strengthen/Premise/Assumption correct answersWhich one of the following, if true, most helps
to strengthen the argument?
Type 2 - Strengthen/Premise/Assumption correct answersWhich one of the following, if true, most
strengthens the argument?
Type 2 - Strengthen/Premise/Assumption correct answersWhich one of the following, if true, adds the
most support for the conclusion of the argument?
Type 2N - Strengthen/Premise/Assumption correct answersWhich one of the following is an assumption
on which the argument relies?
Type 2N - Strengthen/Premise/Assumption correct answersThe reasoning above assumes which one of
the following?
Type 2N - Strengthen/Premise/Assumption correct answersThe relevance of the example of cancer
patients to the argument depends on the assumption that
Type 2N - Strengthen/Premise/Assumption correct answersWhich one of hte following is an assumption
the argument requires in order for its conclusion to be properly drawn?
Type 2S - Strengthen/Premise/Assumption correct answersThe ________ conclusion follows logically if
which one of the following is assumed?